Answer (A) is correct . (200-90)*750-45000-20*750-(750-700)*20
Answer (B) is incorrect because The amount of $27,000 results from including 100 units in ending inventory rather than 50.
Answer (C) is incorrect because The amount of $28,000 results from including 100 units in ending inventory rather than 50 and from failing to write off the $1,000 overhead volume variance.
Answer (D) is incorrect because The amount of $30,000 results from failing to include fixed manufacturing costs in cost of goods sold and failing to subtract ending inventory.